What is the value of S?

What Is The Value Of S?

Answers

Answer 1

The value of S° in the given adjacent angles would be = 26.7°

What are adjacent angles?

Adjacent angles are those angles that are found on the same side of the plane and they share a common vertex.

The adjacent angles are different from the supplementary angles which are angles found in the same side but when measured together sums up to 180°.

The angles 41.6° and S° are two angles that share the same vertex with the sum of 68.3°

Therefore, S° which is the second part of the adjacent angles would be = 68.3+41.6 = 26.7°

Learn more about angles here;

https://brainly.com/question/28394984

#SPJ1


Related Questions

The marks obtained by the students in physics and in mathematics are as follows. Marks in Physics 35 23 47 17 10 43 9 6 28
Marks in Mathematics 30 33 45 23 8 49 12 4 31
Compute of correlation of ranks.
A. 0.2
B. 0.3
C. 0.7
D. 0.9

Answers

The correlation of ranks is approximately 0.2.

Option A is the correct answer.

We have,

To compute the correlation of ranks, we first need to rank the scores in each subject:
Physics: 10, 17, 23, 28, 35, 43, 47
Rank: 1, 2, 3, 4, 5, 6, 7
Mathematics: 4, 8, 12, 23, 30, 31, 33, 45, 49
Rank: 1, 2, 3, 4, 5, 6, 7, 8, 9

Then, we can calculate the differences between the ranks for each student:
Physics ranks: 1-5, 2-3, 3-7, 4-6, 5-1, 6-4, 7-2
Differences: -4, -1, -4, -2, 4, 2, 5
Mathematics ranks: 1-8, 2-6, 3-7, 4-4, 5-1, 6-5, 7-2, 8-3, 9-9
Differences: -7, -4, -4, 0, 4, -1, 5, 5, 0

Next, we can calculate the sum of the products of the differences:
= Sum of products

= (-4)(-7) + (-1)(-4) + (-4)(-4) + (-2)(0) + (4)(4) + (2)(-1) + (5)(5)
= 28 + 4 + 16 + 0 + 16 - 2 + 25
= 87

Finally, we can use the formula for the correlation of ranks:
r = 1 - (6Σd²)/(n(n² - 1))
where d is the difference in ranks and n is the number of scores

Plugging in the values, we get:
r = 1 - (6(87))/(9(81-1))
= 1 - (522)/(648)
= 1 - 0.8056
= 0.1944

= 0.2

Therefore,

The correlation of ranks is approximately 0.2.

Learn more about correlations here:

https://brainly.com/question/31588111

#SPJ11

A man is twice as his son and ten times as old as his grandson. Their combined age amount to 96 years. How old are they?​

Answers

The age of the man , his son, and his grandson is equal to 60 years, 30 years, and  6 years old.

Let x be the age of the son

2x be the age of the man since he is twice as old as his son.

let y be the age of the grandson .

The sum of their ages is 96.

x + 2x + y = 96

Simplifying this equation, we get

⇒3x + y = 96

The man is ten times as old as his grandson,

⇒2x = 10y

Simplifying this equation, we get,

⇒x = 5y

Now substitute x = 5y into the first equation,

⇒3x + y = 96

⇒3(5y) + y = 96

⇒15y + y = 96

⇒16y = 96

⇒y = 6

So the grandson is 6 years old.

Using x = 5y

⇒The son is 30 years old.

Finally, the man is 2x = 2(30)

                                  = 60 years old.

Therefore, the man is 60 years old, his son is 30 years old, and his grandson is 6 years old.

learn more about old here

brainly.com/question/13723647

#SPJ1

Select the table of values where the quadratic function changes direction at a different value of x than the others. A. x -3 -2 -1 0 1 2 3 y 4 3 4 7 12 19 28 B. x -3 -2 -1 0 1 2 3 y 1 -1 1 7 17 31 49 C. x -3 -2 -1 0 1 2 3 y 2 3 2 -1 -6 -13 -22 D. x -3 -2 -1 0 1 2 3 y 28 19 12 7 4 3 4 Reset

Answers

The solution to the given quadratic equation is x= -3 -2 -1 0 1 2 3 y= 2 3 2 -1 -6 -13 -22.

This table of values contains a quadratic function that changes direction at the value of x = 0. This is different from the other tables of values which all have the quadratic function changing direction at the value of x = -2. The y values in this table of values can be described as an upside-down parabola. At x = 0, the y value is -1 and it decreases as x increases to positive values, and it increases as x decreases to negative values.

Therefore correct answer is C. x -3 -2 -1 0 1 2 3 y 2 3 2 -1 -6 -13 -22.

To learn more about the solution of quadratic equation visit:

https://brainly.com/question/18305483.

#SPJ1

Question 7
What is the volume of the pyramid? (Round to the nearest tenth)
11.2 m
11 m
8m

Answers

Answer:

V = 58.7 m^3

Step-by-step explanation:

The formula is V = 1/3 *b*h

where b is the base area

h is the height

From the diagram,

h= 11m

Now we need to find the area of the base.

The base is made of an equilateral triangle, so we know that one of the sides is 8m.

The area of the base is found: 1/2 * a * l

Where a = 8m

l = a/2 = 8m/2 = 4m

So the area of the base is:

b = 1/2 * 8 m* 4m

b = 16m^2

Now plug this into the volume formula:

V = 1/3 b * h

V = 1/3 * 16m^2 * 11m

V = 58.7 m^3

Details Identify the following events as mutually exclusive, independent, dependent or none of these things. You can select more than one option, if appropriate. a) You and a randomly selected student from your class both earn an A in this course. a. Independent b. Dependent c. Mutually Exclusive d. None of these

Answers

For example, if the events were "you earn an A" and "your friend, who always studies with you, earns an A", these events would be dependent because the probability of your friend earning an A would be affected by whether or not you earn an A.

In this case, the events are not mutually exclusive because both events can happen at the same time (i.e., both you and a randomly selected student can earn an A in the course).

The events can be considered independent if one event does not affect the probability of the other event occurring. In this case, whether you earn an A does not affect the probability of the randomly selected student also earning an A. Therefore, the events can be considered independent.

Note that if the events were dependent, it would mean that the probability of one event occurring would affect the probability of the other event occurring. For example, if the events were "you earn an A" and "your friend, who always studies with you, earns an A", these events would be dependent because the probability of your friend earning an A would be affected by whether or not you earn an A.

To learn more about probability visit:

https://brainly.com/question/30034780

#SPJ11

can quite get it can someone help me?!!

Answers

The area of the shaded portion of the given shape is: 114 yd²

What is the area of the shaded region?

The formula for the area of a rectangle is expressed as:

A = L * W

Where:

L is Length

W is Width

Now, to find the area of the shaded part, we will find the area of the bigger rectangle and subtract the area of the unshaded rectangle from it.

Thus:

Area of shaded part = (20 * 12) - (14 * 9)

Area of shaded part = 240 - 126

Area of shaded part = 114 yd²

Read more about area of shaded region at: https://brainly.com/question/1297097

#SPJ1

According to a study on the effects of smoking by pregnant women on rates of asthma in their children, for expectant mothers who smoke 20 cigarettes per day, 22.1% of their children developed asthma by the age of two in the US. A biology professor at a university would like to test if the percentage is lower in another country. She randomly selects 336 women who only deliver one child and smoke 20 cigarettes per day during pregnancy in that country and finds that 70 of the children developed asthma by the age of two. In this hypothesis test, the test statistic, z = and the p-value = (Round your answers to four decimal places.)

Answers

the biology professor cannot conclude that the percentage of children who develop asthma in the new country is lower than the percentage observed in the US study.



The biology professor can use hypothesis testing to determine if the percentage of children who develop asthma in the new country is significantly different from the percentage observed in the US study.

Here are the steps she can take:

1. Define the null and alternative hypotheses:
- Null hypothesis (H0): The percentage of children who develop asthma in the new country is the same as the percentage observed in the US study (i.e., 22.1%).
- Alternative hypothesis (Ha): The percentage of children who develop asthma in the new country is lower than the percentage observed in the US study.

2. Determine the test statistic to use:
- The appropriate test statistic for this scenario is the one-sample proportion z-test.

3. Set the significance level (alpha):
- Let's assume a significance level of 0.05.

4. Calculate the test statistic:
- The sample proportion of children who developed asthma in the new country is p = 70/336 = 0.2083.
- The standard error of the sample proportion is SE = sqrt[(p*(1-p))/n] = sqrt[(0.2083*(1-0.2083))/336] = 0.027.
- The test statistic is z = (p - P) / SE, where P is the proportion observed in the US study. So, z = (0.2083 - 0.221) / 0.027 = -0.463.

5. Determine the p-value and make a decision:
- The p-value is the probability of obtaining a test statistic as extreme or more extreme than the observed value, assuming the null hypothesis is true. Using a standard normal distribution table or calculator, we find that the p-value is 0.3212.
- Since the p-value is greater than the significance level of 0.05, we fail to reject the null hypothesis. There is not enough evidence to conclude that the percentage of children who develop asthma in the new country is significantly different from the percentage observed in the US study.

Therefore, the biology professor cannot conclude that the percentage of children who develop asthma in the new country is lower than the percentage observed in the US study.

Visit to know more about Percentage:-

brainly.com/question/24877689

#SPJ11

8. Is ABC a right triangle? Explain. B 5 A 14 C 9.2​

Answers

Answer: No, it is not.

Step-by-step explanation:

To figure out if a shape is a right triangle, we need to use the pythagorean theorem, which states that a^2 + b^2 = c^2.

In this case, a is equal to 5, b is equal to 9.2, and c is equal to 14.

a^2 is equal to 25 and b^2 is equal to 84.64, we can add these two values together to get 109.64.

Now, we calculate 14^2, which is 196.

We now have something to determine, is 109.64 equal to 196?

Since these two numbers are not equal to each other, the answer is no, and that means this triangle is not a right triangle.

Answer:

Triangle ABC is not a right triangle, as the sum of the squares of the shortest two sides do not equal to the square of the longest side.

Step-by-step explanation:

Pythagoras Theorem explains the relationship between the three sides of a right triangle. The square of the hypotenuse (longest side) is equal to the sum of the squares of the legs of a right triangle:

[tex]\boxed{a^2+b^2=c^2}[/tex]

where:

a and b are the legs of the right triangle.c is the hypotenuse (longest side) of the right triangle.

As we have been given the measures of all three sides of triangle ABC (where AB and AC are the shortest sides, and BC is the longest side), we can use Pythagoras Theorem to determine if the triangle is a right triangle.

If triangle ABC is a right triangle, then AB and AC will be the legs, and BC will be the hypotenuse.

Substitute the values into the formula:

[tex]\implies AB^2+AC^2=BC^2[/tex]

[tex]\implies 5^2+9.2^2=14^2[/tex]

[tex]\implies 25+84.64=196[/tex]

[tex]\implies 109.64=196[/tex]

As 109.64 does not equal 196, triangle ABC is not a right triangle.

Tiffany wants to buy the car from her mother now.(t=5)A fair price for the car will be $ in 5 years

Answers

A) The correct features is,

⇒ a = 9000

⇒ r = 15%

B) The equation correctly models the context of the problem is,

⇒ y = 9000 (0.85)ˣ

We have to given that;

Tiffany’s mother bought a car for $9000 five years ago.

And, She wants to sell it to Tiffany based on a 15% annual rate of depreciation.

Now, We have;

the exponential growth formula is,

⇒ y = a(1 − r)ˣ

Here, We have;

⇒ a = 9000

⇒ r = 15%

Thus, We get;

The equation correctly models the context of the problem is,

⇒ y = a(1 − r)ˣ

⇒ y = 9000 (1 - 0.15)ˣ

⇒ y = 9000 (0.85)ˣ

Therefore, We get;

A) The correct features is,

⇒ a = 9000

⇒ r = 15%

B) The equation correctly models the context of the problem is,

⇒ y = 9000 (0.85)ˣ

Learn more about the percent visit:

brainly.com/question/24877689

#SPJ1

Complete question is,

Tiffany’s mother bought a car for $9000 five years ago. She wants to sell it to Tiffany based on a 15% annual rate of depreciation.

Part A. Identify each feature of the problem as it relates to the context and the exponential growth formula: y=a(1−r)t

a=

r=

Part B. Which equation correctly models the context of the problem?

Choose : A. y=9000(0.15)t

or B. y=9000(0.85)t

Answer : The equation is

Part C.

Tiffany wants to buy the car from her mother now. (t = 5)

A fair price for the car will be about $

in 5 years.

Confidence Interval Calculation.
1. You randomly sample beetles from the Smith Island Population. For a sample of size 20, the sample mean weight is 0.21 grams. You know the colony population standard deviation in weight is 0.05 grams. Find the 95% confidence interval for the population mean. Set up the equation, solve to an upper and lower limit and write out the correct confidence interval statement. Assume the population is normally distributed and the critical z value you will need is 96

Answers

We are 95% confident that the true population mean weight of beetles in the Smith Island population lies between 0.1876 grams and 0.2324 grams.

To calculate the 95% confidence interval for the population mean, we can use the formula:

CI = X ± z*(σ/√n)

where X is the sample mean, σ is the population standard deviation, n is the sample size, and z* is the critical value of the standard normal distribution corresponding to the desired level of confidence.

In this case, X = 0.21 grams, σ = 0.05 grams, n = 20, and the critical z value for a 95% confidence level is 1.96.

So, the confidence interval can be calculated as:

CI = 0.21 ± 1.96*(0.05/√20)

= 0.21 ± 0.0224

Therefore, the 95% confidence interval for the population mean weight of beetles in the Smith Island population is (0.1876, 0.2324) grams.

The correct confidence interval statement would be: We are 95% confident that the true population mean weight of beetles in the Smith Island population lies between 0.1876 grams and 0.2324 grams.

To learn more about population visit:

https://brainly.com/question/24786731

#SPJ11

A coin is tossed 4 times. What are the odds against the coin showing heads all 4 times?

Answers

The odds against the coin showing heads all 4 times its tossed is 15:1.

Explaining how to get the odd of a tossed coin

Probability of getting heads on one toss of a fair coin is 1/2

Since the coin is tossed four times, the probability of getting heads all four times is:

P(H) =  (1/2) x (1/2) x (1/2) x (1/2) = 1/16.

Recall that the odds against an event happening are the ratio of the number of ways it can't happen to the number of ways it can happen.

In this case, the number of ways the coin won't show heads all four times is:

P(T) = 15 (there are 16 possible outcomes and only one of them is all heads). Therefore, the odds against the coin showing heads all four times are 15 to 1.

Learn more about probability here:

https://brainly.com/question/27742286

#SPJ1

From question 1, recall the following definition. Definition. An integer n leaves a remainder of 3 upon division by 7 if there exists an integer k such that n = 7k + 3. (a) Show that the integer n = 45 leaves a remainder of 3 upon division by 7 by verifying the definition above. (b) Show that the integer n = -32 leaves a remainder of 3 upon division by 7 by verifying the definition 3 above. (c) Show that the integer n = 3 leaves a remainder of 3 upon division by 7 by verifying the definition (d) Show that the integer n= -4 leaves a remainder of 3 upon division by 7 by verifying the definition а (e) Use a proof by contradiction to prove the following theorem: Theorem. The integer n = 40 does not leave a remainder of 3 upon division by 7.

Answers

This contradicts our assumption, so we conclude that 40 does not leave a remainder of 3 upon division by 7.

(a) To show that 45 leaves a remainder of 3 upon division by 7, we need to find an integer k such that 45 = 7k + 3. We can write 45 as 42 + 3, which gives us 45 = 7(6) + 3. Thus, n = 45 satisfies the definition and leaves a remainder of 3 upon division by 7.

(b) To show that -32 leaves a remainder of 3 upon division by 7, we need to find an integer k such that -32 = 7k + 3. We can write -32 as -35 + 3, which gives us -32 = 7(-5) + 3. Thus, n = -32 satisfies the definition and leaves a remainder of 3 upon division by 7.

(c) To show that 3 leaves a remainder of 3 upon division by 7, we need to find an integer k such that 3 = 7k + 3. We can write 3 as 0 + 3, which gives us 3 = 7(0) + 3. Thus, n = 3 satisfies the definition and leaves a remainder of 3 upon division by 7.

(d) To show that -4 leaves a remainder of 3 upon division by 7, we need to find an integer k such that -4 = 7k + 3. We can write -4 as -7 + 3, which gives us -4 = 7(-1) + 3. Thus, n = -4 satisfies the definition and leaves a remainder of 3 upon division by 7.

(e) To prove that 40 does not leave a remainder of 3 upon division by 7, we assume the opposite, that is, we assume that 40 does leave a remainder of 3 upon division by 7. This means that there exists an integer k such that 40 = 7k + 3. Rearranging this equation gives us 37 = 7k, which means that k is not an integer, since 37 is not divisible by 7. This contradicts our assumption, so we conclude that 40 does not leave a remainder of 3 upon division by 7.

To learn more about remainder visit:

https://brainly.com/question/20286167

#SPJ11

"The diameters of Ping-Pong balls manufactured at a
large factory are normally distributed with a mean of 3cm and a
standard deviation of 0.2cm. The probability that a randomly
selected Ping-Pong ball has a diameter of less than 2,7 cm is"

Answers

The probability that a randomly selected Ping-Pong ball has a diameter of less than 2.7 cm is approximately 0.0668 or 6.68%.

To solve this problem, we need to standardize the value of 2.7 using the formula:

z = (x - μ) / σ

where:

x = 2.7 (the value we want to find the probability for)

μ = 3 (mean)

σ = 0.2 (standard deviation)

z = (2.7 - 3) / 0.2

z = -1.

Now, we need to find the probability that a randomly selected Ping-Pong ball has a diameter of less than 2.7 cm, which is the same as finding the area to the left of z = -1.5 on the standard normal distribution curve. We can use a standard normal distribution table or a calculator to find this area.

Using a standard normal distribution table, we can find the area to the left of z = -1.5 is 0.0668.

Therefore, the probability that a randomly selected Ping-Pong ball has a diameter of less than 2.7 cm is approximately 0.0668 or 6.68%.

To learn more about probability visit:

https://brainly.com/question/15124899

#SPJ11

a chi-square goodness-of-fit test was conducted to determine whether the data provide convincing evidence that the distribution has changed. the test statistic was 10.13 with a p-value of 0.0175. which of the following statements is true?

Answers

To know chi-square goodness-of-fit test conducted to determine whether the data provide convincing evidence that the distribution has changed. The test statistic was 10.13 with a p-value of 0.0175.

To ascertain if the observed data adheres to a predetermined distribution, the chi-square goodness-of-fit test is utilised.

The test statistic is determined using the following formula: 2 = [(O - E)2 / E]where 2 is the test statistic, is the sum of all the categories, and O and E are the observed and predicted frequencies.

If the null hypothesis is true, the p-value is the likelihood that a test statistic will be equally extreme or more extreme than the observed one.

The null hypothesis in this situation is that the distribution has not altered.

If the p-value is less than 0.05, we reject the null hypothesis and come to the conclusion that there is a statistically significant difference between the observed and predicted frequencies, indicating that the distribution has really changed. This is because the generally used significance level is 0.05.

The test statistic in this instance is 10.13, and the p-value is 0.0175.

We reject the null hypothesis since the p-value is less than 0.05 and come to the conclusion that the data is strong evidence that the distribution has altered.

Learn more about chi-square goodness-of-fit test:

https://brainly.com/question/16865619

#SPJ11

Type the correct answer in each box. Spell all the words correctly, and use numerals instead of words for numbers. If necessary, use / for the fraction bar(s). Two shaded triangles are graphed in an x y plane. The vertices are as follows: first: A (8, 8), B (10, 4), and C (2, 6); second: A prime (6, negative 8), B (8, negative 4), and C (0, negative 6). We can show that ∆ABC is congruent to ∆A′B′C′ by a translation of 2 unit(s) and a across the -axis.

Answers

We can show that ∆ABC is congruent to ∆A′B′C′ by a translation of (x-2, y) unit(s) and a across the x-axis.

The coordinates of the triangle are A(8, 8), B(10, 4), C(2, 6), while the triangle A'B'C' is at A'(6, -8), B'(8, -4), C'(0, -6).

If a point O(x, y) is translated a units on the x axis and b units on the y axis, the new coordinate is O'(x+a, y+b).

If a point O(x, y) is reflected across the x axis, the new coordinate is O'(x, -y)

Hence if triangle ABC is translated -2 units on the x axis (2 units left), the new coordinates are A*(6, 8), B*(8, 4), C*(0, 6). If a reflection across the x axis is then done, the new coordinates are A'(6, -8), B'(8, -4), C'(0, -6).

Learn more about coordinate triangle: brainly.com/question/2644832

#SPJ1

Help need answers!!! 100 POINTS!!! what does x equal, and what does angle m

Answers

Answer:

x = 10.1, AXY = 71.7 degrees

Step-by-step explanation:

There are two ways to solve this problem. You could either do 7x+1+108.3=180 or 180-108.3, then take that number and set it equal to 7x+1.

I will be using the latter. (You can do this because angle YXB is a linear pair with angle AXY. This means they add up to 180. So to find angle AXY, you subtract 180 from 108.3)

[tex]180-108.3=71.7\\\\7x+1=71.7\\\\[/tex]

Subtract one from each side to move variables to the left and constants to the right.

[tex]7x+1-1=71.7-1\\\\7x=70.7[/tex]

Divide seven by both sides to isolate the variable.

[tex]\frac{7x}{7}=\frac{70.7}{7} \\\\x=10.1[/tex]

So now we know what x is. So to find AXY, you substitute it back into the equation.

[tex]7(10.1)+1=71.7\\\\70.1+1=71.7?\\\\71.1=71.7?[/tex]

The diameter of a circle is 18 yards. What is the circle's circumference? Use 3.14 for ​.

Pls will give brainliest
20 points

Answers

Answer:

The answer is 56.52 .

Step-by-step explanation:

18 multiple 3.14

What is the best way to display data if there is a narrow range, and we want to see the shape of the data?

Multiple Choice

A. frequency table with intervals

B. frequency table

C. stem-and-leaf plot

D. line plot

Answers

Answer:

When data has a narrow range and we want to see the shape of the data, the best way to display the data is through a line plot. A line plot is a graphical display of data that uses dots placed above a number line to show the frequency of values in a set of data. It is useful for showing the distribution of a small set of data, especially when the data has a narrow range of values. The line plot allows us to see how many times each value occurs and to identify the mode(s) of the data.

When data has a narrow range and we want to see its shape, a stem-and-leaf plot is the best way to display it. In a stem-and-leaf plot, the data is divided into two parts: the stem and the leaf. The stem is the leftmost digit(s) of each data point, and the leaf is the rightmost digit. This plot allows us to quickly see the distribution of the data and identify any outliers or patterns.

What is the least common multiple (LCM) of xy, x^2, and xy-? X Xy^2
Ax
bxy
cx^2y^2
dx^4y^3​

Answers

The answer is option [tex](cx^2y^2).[/tex]

What is  least common multiple (LCM) of xy, x^2, and xy-? X Xy^2

To find the least common multiple (LCM) of [tex]xy, x^2,[/tex] and xy^2, we need to factor each term into its prime factors and then take the highest power of each factor.

xy = (x) * (y)

x^2 = (x) * (x)

xy^2 = (x) * [tex](y^2)[/tex]

The prime factorization of the given terms are:

xy = (x) * (y)

x^2 = (x) * (x)

xy^2 = (x) * [tex](y^2)[/tex]

So, the LCM can be found by taking the highest power of each factor, which gives us:

LCM = [tex](x^2)[/tex] * [tex](y^2)[/tex] =[tex]x^2y^2[/tex]

Therefore, the answer is option [tex](cx^2y^2).[/tex]

learn more about LCM

brainly.com/question/20739723

Patricia is studying a polynomial function f(x). Three given roots of f(x) are Negative 11 minus StartRoot 2 EndRoot i, 3 + 4i, and 10. Patricia concludes that f(x) must be a polynomial with degree 4. Which statement is true?

Answers

The statement that is true is that D. Patricia is not correct because both 3 – 4i  and 11+√2i  must be roots.

What are polynomial function?

A polynomial function is a function that involves only non-negative integer powers or only positive integer exponents of a variable in an equation like the quadratic equation, cubic equation, etc.

From the information, Patricia is studying a polynomial function f(x). Three given roots of f(x) are -11-√2i , 3 + 4i, and 10. Patricia concludes that f(x) must be a polynomial with degree 4.

In this case, the correct option is D.

Learn more about polynomial on

https://brainly.com/question/4378934

#SPJ1

Patricia is studying a polynomial function f(x). Three given roots of f(x) are -11-√2i , 3 + 4i, and 10. Patricia concludes that f(x) must be a polynomial with degree 4. Which statement is true?

A. Patricia is correct because -11+√2i must be a root.

B. Patricia is correct because 3 – 4i must be a root.

C. Patricia is not correct because both 3 – 4i  and -11+√2i  must be roots.

D. Patricia is not correct because both 3 – 4i  and 11+√2i  must be roots

Answer: D

Step-by-step explanation:

edge 2023

Given the logistic function 3 x(t) = e-1.08 t +0.09 The time needed to reach x(t)= 98 is t=3. Select one: a. True b. False

Answers

The statement "Given the logistic function 3 x(t) = e-1.08 t +0.09 The time needed to reach x(t)= 98 is t=3." is :

(b) False

The logistic function is a mathematical function that is used to model growth processes that are limited by saturation. It is often used in the field of biology to model population growth, as well as in economics to model the growth of markets and the adoption of new technologies.

Given the logistic function x(t) = 3e^(-1.08t) + 0.09, you want to determine if x(t) = 98 when t = 3.

Step 1: Plug in t = 3 into the function
x(3) = 3e^(-1.08*3) + 0.09

Step 2: Calculate the result
x(3) ≈ 3e^(-3.24) + 0.09 ≈ 0.0705

Since x(3) ≈ 0.0705 and not 98, the statement "The time needed to reach x(t) = 98 is t = 3" is false. Therefore, the correct answer is:

b. False

To learn more about the logistic function visit : https://brainly.com/question/29312525

#SPJ11

the ratio of students who prefer pineapple to students who prefer kiwi is 12 to 5. which pair of equivalent ratios could be used to find how many students prefer kiwi if there are 357 total students

Answers

To find out how many students prefer Kiwi when there are 357 total students, we can use the equivalent ratios of 5:12 or 12:5.

The ratio of students who prefer pineapple to students who prefer kiwi is given as 12 to 5, which means that for every 12 students who prefer pineapple, 5 students prefer kiwi. We can represent this ratio as 12:5.

To find out how many students prefer kiwi, we need to determine the proportion of the total number of students that prefer kiwi. Since the total number of students is 357, we can set up a proportion with the ratio of students who prefer Kiwi to the total number of students. Using the equivalent ratio of 5:12, we can set up the proportion as follows:

5/12 = x/357

Here, x represents the number of students who prefer Kiwi. To solve for x, we can cross-multiply and simplify the proportion as follows:

5 * 357 = 12 * x
1785 = 12x
x = 1785/12
x = 148.75

Since we cannot have a fractional number of students, we need to round our answer to the nearest whole number. Therefore, we can conclude that approximately 149 students prefer Kiwi out of a total of 357 students.

To learn more about Proportions, visit:

https://brainly.com/question/870035

#SPJ11

7. Dr. Agoncillo is an orthopedic surgeon. He spent 4 years in undergrad, 4 years in
medical school, 5 years of residency, and completed a 1-year fellowship to
specialize in treating foot and ankle injuries. How many years total did Dr. Agoncillo
complete of post-secondary education?
8. Dan wants to stay hydrated for marching band practice. He drank two 20-ounce
bottles of Gatorade, three 16-ounce water bottles, and 1 large 32-ounce Bojangles
sweet tea. How many total fluid ounces did Dan consume?
9. The physical therapy clinic has 27 double 6-inch ACE wraps, 43 single 3-inch wraps,
93 single 6-inch ACE wraps, and 12 2-inch ACE wraps. How many ACE wraps in all
are in stock at this physical therapy clinic?
10. Karen is a hungry teenager and her favorite snack after school is one regular-size
Snickers® bar (20 grams of sugar, 11 grams of fat, 3 grams of protein), one small
bag of Doritos (1 gram of sugar, 8 grams of fat, 2 grams of protein), and one can of
Mt. Dew® (46 grams of sugar, 0 grams of fat, 0 grams of protein). How many total
grams of sugar, fat, and protein did Karen consume in this snack?

Answers

Answer:

7. Dr. Agoncillo completed a total of 14 years of post-secondary education. (4 years undergrad + 4 years medical school + 5 years residency + 1 year fellowship = 14 years)

8. Dan consumed a total of 124 fluid ounces. (2 x 20 + 3 x 16 + 1 x 32 = 40 + 48 + 32 = 120 fluid ounces)

9. There are 175 ACE wraps in stock at this physical therapy clinic. (27 x 2 + 43 x 1 + 93 x 1 + 12 x 1 = 54 + 43 + 93 + 12 = 175 ACE wraps)

10. Karen consumed a total of 67 grams of sugar, 19 grams of fat, and 5 grams of protein in this snack. (Snickers: 20g sugar + 11g fat + 3g protein = 34g total; Doritos: 1g sugar + 8g fat + 2g protein = 11g total; Mt. Dew: 46g sugar + 0g fat + 0g protein = 46g total; 34g + 11g + 46g = 91g total sugar; 11g + 0g + 0g = 11g total fat; 3g + 2g + 0g = 5g total protein)

Step-by-step explanation:

Mark Brainliest!!

A market has 3,000 oranges. If the market has 100 fruit crates and wants to put the same number of oranges in each crate, how many oranges will go into each crate?

Answers

Answer:

30 oranges

Step-by-step explanation:

Divide 3,000 by 100 and you get the number of 30 so which means they can put 30 oranges each box if they wanted to.

Step-by-step explanation:

Answer: 30

Step-by-step explanation:

divide 3000 by 100 and then you git your answer

Tell which one is true and why. 1-Having x2 f(x) = g(x) = x + 1 r - 1 and the equality f(x) = g(x), about the functions f and g we can say: A) The functions f and g are the same B) Only the expressions of fand g are the same C) The data did not allow whether or not f and g are equal, D) The functions f and g are not the same

Answers

The functions f(x) and g(x) given that [tex]x^2 f(x) = g(x) = x + 1[/tex], the function f and g are not the same, option D.

Rewriting the equation

We are given [tex]x^2  f(x) = g(x) = x + 1[/tex]. Let's rewrite this as two separate equations:

[tex]x^2 f(x) = x + 1[/tex]

g(x) = x + 1

Determining the relationship between f(x) and g(x)

We can rearrange the first equation to solve for f(x):

[tex]f(x) = (x + 1) / x^2[/tex]

Now, we have expressions for both f(x) and g(x):

[tex]f(x) = (x + 1) / x^2[/tex]

[tex]g(x) = x + 1[/tex]

Comparing the expressions for f(x) and g(x), we can see that they are not the same. The expressions for f(x) and g(x) differ, so the functions f(x) and g(x) are not the same.

Therefore, the correct answer is D) The functions f and g are not the same.

To know more about functions: https://brainly.com/question/11624077

#SPJ11

Assume that two fair dice are rolled. Define two events as follows:
F = the total is five
E = an odd total shows on the dice
a. Compute P(F) and
b. Compute P(F|E). Explain why one would expect the probability of F to change as it did when we added the condition that E had occurred.

Answers

When two fair dice are rolled,

(a) P(F) = 1/9

(b) P(F|E) = 1/5

a. To compute P(F), we need to find the probability that the total of two dice is five. There are four ways to obtain a total of five: (1,4), (2,3), (3,2), and (4,1). Since each die has six possible outcomes, there are 6x6=36 possible outcomes when two dice are rolled. Therefore, P(F) = 4/36 = 1/9.

b. To compute P(F|E), we need to find the probability that the total of two dice is five given that the total is odd. Since the sum of two odd numbers is always even, we know that if an odd total shows on the dice, then the sum must be either 3, 5, 7, 9, or 11. Out of these possibilities, only one yields a total of 5, which is (2,3). Therefore, P(F|E) = 1/5.

We would expect the probability of F to change when we condition on E because the occurrence of E affects the sample space. When we know that an odd total shows on the dice, we can eliminate some of the possible outcomes and reduce the sample space. This makes it more likely that the remaining outcomes will satisfy the condition for F, which increases the probability of F. Therefore, P(F|E) is greater than P(F) because E provides additional information that makes F more likely.

To learn more about probability visit : https://brainly.com/question/13604758

#SPJ11

Draw the Voronoi tile for the following data set (-1,-1), (-1,1), (1,-1), (1,1),(0,0) around the Assume that the point(0,0) has a classification and all the other points have a + classification. If we use 1-nearest neighbors, what will be probability of a point chosen uniformly at random in the region -1 5xs1,-15 y s 1 to be classified as '+'? [5 Marks)

Answers

The probability of a point chosen uniformly at random in the region [tex]-1\leq x\leq1[/tex] and [tex]-1\leq y\leq1[/tex]  to be classified as '+' is [tex](\frac{4 - A_{minus}}{4})[/tex].

To draw the Voronoi tile for the given data set and find the probability of a point being classified as '+', follow these steps:

1. Plot the data points: Plot the points (-1,-1), (-1,1), (1,-1), (1,1), and (0,0) on a graph. Label (0,0) as '-' and the other points as '+'.

2. Construct the Voronoi diagram:

For each pair of neighboring '+' points, draw a line that is equidistant from both points and bisects the line connecting them. These lines will divide the space into regions called Voronoi tiles, where each tile contains one data point, and any point within that tile is closer to the data point it contains than to any other data point.

3. Identify the tile containing the '-' point:

In this case, the Voronoi tile surrounding (0,0) will be the region that is closer to the '-' point than to any '+' point.

4. Calculate the area of the Voronoi tile containing the '-' point:

Since we are considering the region [tex]-1\leq x\leq1[/tex] and [tex]-1\leq y\leq1[/tex], find the area of the intersection of this region with the Voronoi tile containing the '-' point.

5. Calculate the total area of the considered region: The total area of the considered region is

(-1 to 1)(-1 to 1) = 2(2) = 4 square units.

6. Determine the probability of a point being classified as '+':

The probability of a point chosen uniformly at random in the considered region being classified as '+' is equal to the ratio of the area of the region not covered by the '-' Voronoi tile to the total area of the considered region.

Let's say the area of the '-' Voronoi tile is [tex]A_{minus}[/tex]. Then, the probability of a point being classified as '+' is [tex](\frac{4 - A_{minus}}{4})[/tex].


To know more about the  Voronoi diagram visit:

https://brainly.com/question/29264749

#SPJ11

Find the linear approximation of the given function at ( Pi, 0). F(x,y)= square root y +(cos(x))^2 F(x,y)=

Answers

The linear approximation of F at (Pi, 0) is [tex]-Pi^2cos^2(Pi).[/tex]

To discover the linear approximation of the given function at (Pi, 0), we need to first discover the partial derivatives of the function with respect to x and y evaluated at (Pi, zero).

Partial derivative of F with recognize to x:

∂F/∂x = -2sin(x)cos(x)

evaluated at (Pi, 0):

∂F/∂x(Pi, 0) = -2sin(Pi)cos(Pi) = 0

Partial derivative of F with recognize to y:

∂F/∂y = 1/(2√y)

evaluated at (Pi, 0):

∂F/∂y(Pi, 0) = 1/(2√0) = undefined

For the reason that partial derivative of F with respect to y is undefined at (Pi, 0), we can't use the multivariable Taylor collection to discover the linear approximation. as an alternative, we will use the formula for the linear approximation:

[tex]L(x,y) = f(a,b) + ∂f/∂x(a,b)(x-a) + ∂f/∂y(a,b)(y-b)[/tex]

Wherein (a,b) is the factor at which we want to find the linear approximation.

In this case, a = Pi and b = 0. So, the linear approximation is:

[tex]L(x,y) = F(Pi, 0) + ∂F/∂x(Pi, 0)(x - Pi)[/tex]

[tex]L(x,y) = sqrt(0) + (cos(Pi))^2(0 - Pi)[/tex]

[tex]L(x,y) = -Pi^2cos^2(Pi)[/tex]

Consequently, the linear approximation of F at (Pi, 0) is [tex]-Pi^2cos^2(Pi).[/tex]

Learn more about  linear approximation :-

https://brainly.com/question/28384323

#SPJ4

I need help with this question!

Answers

The perimeter of the figure in this problem is given as follows:

P = 36.6.

What is the perimeter of a polygon?

The perimeter of a polygon is given by the sum of all the lengths of the outer edges of the figure, that is, we must find the length of all the edges of the polygon, and then add these lengths to obtain the perimeter.

For this problem, three of the lengths are quite straightforward, as follows:

10, 4 and 10.

The fourth length is half the circumference of a circle of diameter 4 = radius 2, hence it is given as follows:

C = 2πr

C = 4π.

C = 12.6.

Hence the perimeter of the figure is given as follows:

P = 10 + 4 + 10 + 12.6

P = 36.6.

More can be learned about the perimeter of a polygon at https://brainly.com/question/3310006

#SPJ1

Where do erasers go for vacation? missing lengths

Answers

Answer: ''pencil veinya''    (Pennsylvania)

Step-by-step explanation:

Other Questions
The functional unit of a skeletal muscle fiber is the A. SarcomereB. MyofibrilC. Sarcoplasmic reticulumD. SarcomereE. Myofilament part a. When determining the value of asset, what are some issues or events that need to be considered? (Think about both short-term and long-term assets.)part b . Provide examples of items beyond the listed purchase price that may need to be considered in the value (or acquisition cost) of an asset.part c. When working with assets (either short or long-term), why do you think management is given the option to choose a method of accounting that affects the asset value? Give an example of an accounting method choice that affects the value of an asset. Why did Athenian boys have to have a good memory? why it is essential in thermocouple-extension wire junction to do connect carefully to avoid large errors? Let V be an n-dimensional vector space with ordered basis , and . Let C be the change of coordinate matrix from to , B bethe change of coordinate matrix from to and A be the change which of the following describes a program? several projects that address the same opportunity several activities that are recurring group of work with well-defined results has a specific budget and end date 1. Identify ONE claim made in the passage"The seventeenth century saw the heyday of the [European] East India Companies. They became an inevitable part of the politics and economy of South India, finding their way into the society and even the vocabulary of the local people. The kumbini,* as the local populace called it, was not to be ignored. As with the rest [of the population], the spinners and weavers, the washers, the dyers and the once powerful textile-merchant guilds had their links with the company [warehouses]. Of course there were also those merchants and weavers who operated independently of the company, but their proportion was small. With the establishment of the rival European companies in the seventeenth century, the lives of the weavers [of South India] no longer revolved around the temple but around the European [warehouses] and the towns [on the outskirts of European trading posts]. The weavers gradually began losing their bargaining power and independence. . . . Their creativity also suffered since they were compelled to copy mechanically the [designs] provided to them [by the companies officials]. Economically they were much worse off. "*a Tamil pronunciation of the word "company"Vijaya Ramaswamy, Indian historian, book published in 1985 Question 5What are the mandatory marketing tactics? an investor with $10,000 available to invest has the following options: (1) he can invest in a risk-free savings account with a guaranteed 3% annual rate of return; (2) he can invest in a fairly safe stock, where the possible annual rates of return are 6%, 8%, or 10%; or (3) he can invest in a more risky stock, where the possible annual rates of return are 1%, 9%, or 17%. the investor can place all of his available funds in any one of these options, or he can split his $10,000 into two $5,000 investments in any two of these options. the joint probability distribution of the possible return rates for the two stocks is given in the file p09 34.xlsx. Dorsal venous arch collects blood from the ______ one of the 20 codis core loci strs used in examining noncoding dna sequences is d1s1656 and has the sequence (gata)n. from a statistical approach, how often would you expect this dna sequence to appear in a noncoding region of human dna that has relatively equal distributions of the four nucleotides (i.e., it is not g/c or a/t rich)? Crane Clark Opened Cranes Cleaning Service On July 1, 2022. During July, The Following Transactions Were Completed. July 1 Crane Invested $19,900 Cash In The Business. 1 Purchased Used Truck For $8,800, Paying $3,900 Cash And The Balance On Account. 3 PurchasedCrane Clark opened Cranes Cleaning Service on July 1, 2022. During July, the following transactions were completed. July 1Crane invested $19,900 cash in the business.1Purchased used truck for $8,800, paying $3,900 cash and the balance on account.3Purchased cleaning supplies for $2,000 on account.5Paid $1,800 cash on a 1-year insurance policy effective July 1.12Billed customers $4,500 for cleaning services.18Paid $1,600 cash on amount owed on truck and $1,500 on amount owed on cleaning supplies.20Paid $2,500 cash for employee salaries.21Collected $3,400 cash from customers billed on July 12.25Billed customers $6,000 for cleaning services.31Paid $350 for the monthly gasoline bill for the truck.31Withdraw $5,600 cash for personal use.MULTIPLE PART QUESTION, Please answer a2, b+c, d3, f1, f2, and g.a2:Post the July transactions. (Post entries in the order of journal entries posted above.)CashNo. 101DateExplanationDebitCreditBalanceb+c:Prepare a trial balance at July 31 on a worksheet. Enter the following adjustments on the worksheet and complete the worksheet.(1)Unbilled and uncollected revenue for services performed at July 31 were $2,700.(2)Depreciation on equipment for the month was $500.(3)One-twelfth of the insurance expired.(4)An inventory count shows $600 of cleaning supplies on hand at July 31.(5)Accrued but unpaid employee salaries were $1,010.CRANES CLEANING SERVICEWorksheetchoose the accounting period For the Month Ended July 31, 2022For the Year Ended July 31, 2022July 31, 2022Trial BalanceAdjustmentsAdjusted Trial BalanceIncome StatementBalance SheetAccount TitlesDr.Cr.Dr.Cr.Dr.Cr.Dr.Cr.Dr.Cr.d3:Prepare a classified balance sheet at July 31. (List Current Assets in order of liquidity.)f1:Journalize closing entries and complete the closing process. (Credit account titles are automatically indented when amount is entered. Do not indent manually. Record journal entries in the order displayed in the problem.)DateAccount Titles and ExplanationDebitCreditf2:Post closing entries. (Post entries in the order of journal entries posted above.)CashNo. 101DateExplanationDebitCreditBalanceg:Prepare a post-closing trial balance at July 31.CRANES CLEANING SERVICEPost-Closing Trial Balancechoose the accounting period For the Year Ended July 31, 2022July 31, 2022For the Month Ended July 31, 2022DebitCredit the maximum tailwind component of the airplane is 10 knots. the actual tailwind calculated is 11 knots. other aircraft are continuing to land, so you decide to ignore the limitation and land as well. which hazardous attitude are you displaying? T/F An efficient market has many impediments to the free flow of goods and services. You want to purchase a new bike that costs $125.99. To save for the bike, you mow your neighbor's yard for $40 each week. How much money will you have left over if you buy the bike after saving for 4 weeks? 50 Points! Answer Correctly Please!!!In its ruling on the Civil Rights Cases of 1883, the Supreme Court stated thatA) The law applied to other minorities besides African Americans.B) The law banned racial discrimination by the government, not by individuals.C) Civil rights included the freedom to use all public facilities.D) The denial of equal accommodations for African Americans was similar to slavery. A spinner is divided into three sections: red, blue, and green. The red section is 2/5 of the area of the spinner. The blue section is 1/2 of the area of the spinner. Give the probability for each outcome. Express your answers as fractions. Use the given circumference to find the surface area of the spherical object. a pincushion with C = 24 cm Create your own problem using physical quantities on inverse variation a i Design a formula using the concepts of variation I and solve that problem and state conclusion] in the family life-cycle stage of families with adolescents, they are faced with development of increasing ____ for adolescents, midlife reexamination of ____ and ____ issues, and initial shift towards concern for the _____ generation.